Intégrale, le retour ! - Page 2 — Les-mathematiques.net The most powerful custom community solution in the world

Intégrale, le retour !

2

Réponses

  • J'ai oublié le $x^2$ c'est bête. :-D
    Je corrigerai plus tard en donnant la fin de la résolution.
  • Je n'avais pas pensé à attaquer la question sous l'angle des séries (quand tu as une idée qui marche en tête c'est difficile de prendre du recul, d'où ma question).
  • Pas sûr que mes séries soient une si bonne idée finalement. Si on reprend sans fautes, on a $$\int_0^1 \frac{\arctan(x^2)}{1+x^2} dx =\sum_{k=0}^{+\infty} \frac{(-1)^k}{2k+1} \int_0^1 \frac{x^{4k+2}}{1+x^2} dx = \sum_{k=0}^{+\infty} \frac{(-1)^k}{2k+1} \sum_{j=0}^{+\infty} (-1)^j \int_0^{1} x^{4k+2} x^{2j}.$$ Au total l'intégrale se réécrit $$\sum_{j,k=0}^{+\infty} \frac{(-1)^{j+k}}{(2k+1)(4k+2j+3)}.$$ On la décomposition $$\frac{1}{(2k+1)(4k+2j+3)} = \frac{1}{(2k+1)(2j+1)} -\frac{2}{(2j+1)(4k+2j+3)}.$$ Donc on peut trouver $$\sum_{j,k =0}^{+\infty} \frac{(-1)^{j+k}}{(2k+1)(4k+2j+3)} = \frac{\pi^2}{16} -2 \sum_{j,k =0}^{+\infty} \frac{(-1)^{j+k}}{(2j+1)(4k+2j+3)}.$$ Les deux séries que l'on voit apparaître sont très semblables. Il faudrait exprimer l'une en fonction de l'autre pour conclure.
  • Bon, en réalité la décomposition évoquée plus haut est exactement équivalente à une intégration par partie. On a $$\int_0^1 \frac{\arctan(x^2)}{1+x^2} dx = \left[\arctan(x)\arctan(x^2) \right]_0^1 -2 \int_0^1 \frac{x\arctan(x)}{1+x^4}dx.$$
  • Allez, une dernière méthode, qui me semble en réalité la plus raisonnable. Je poursuis mon raisonnement après l'intégration par partie. Considérons $$I(a) = \int_0^1 \frac{x\arctan(ax)}{1+x^4} dx.$$ On a $$I'(a) = \int_0^1 \frac{x^2}{(1+a^2 x^2)(1+x^4)} dx = \frac{1}{a^2} \left( \int_0^1 \frac{1}{1+x^4} dx - \int_0^1 \frac{1}{(1+a^2 x^2)(1+x^4)} dx \right).$$ A partir de là, on casse la grosse fraction de la section intégrale en fraction simple et tout peut se calculer avec la méthode habituelle. Reste à primitiver par rapport à $a$ (c'est faisable car il n'y aura que des polynomes en $a$ ) et évaluer en $1$ puis assembler tous les bouts.
  • Je suis sceptique sur la faisabilité. J'ai cru que les choses s'arrangeaient mais j'en suis moins sûr maintenant.

    https://www.wolframalpha.com/input/?i=integrate+1/(1+a^2*x^2)*1/(1+x^4),x=0,1
  • Wolfram ne sait pas le faire d'un coup. Mais si tu casses en deux, tu verras qu'il te donnera une réponse pour chaque bout.
  • En parlant d'Arctangente je suis entrain de travailler sur une intégrale sauriez vous m'aider ?
    $$\frac1\pi \int_0^{\pi } \arctan\left(\tan \left(\frac{t-\sin t+\frac{\pi }{2}}2\right)\right) \, dt+\frac{1}{\pi }$$

    Merci d'avance et désolé Fdp si je me permet de poster dans ton fil .
  • Le problème est comment tu calcules les intégrales définies qui en résultent dont les intégrandes sont des fonctions de $a$.

    Sauf erreur, de ma part, il va falloir calculer $\displaystyle \int_0^1 \frac{a\arctan a}{1+a^4}\,da$

    https://www.wolframalpha.com/input/?i=integrate+1/a^2*1/(1+a^2*x^2)*1/(1+x^4),x=0,1

    J'avais oublié de diviser par $a^2$.

    et puis, sauf erreur, il y a le problème d'intégrer $\frac{1}{a^2}$ pour $a$ variant de $0$ à $1$
  • Noel:

    Rien n'est à moi sur ce forum, je me suis seulement permis de poser une question qui , j'espère, peut avoir un intérêt et les réponses apportées ou tentées ont un intérêt aussi.

    PS:
    Composer la fonction arctan et la fonction tan complique les choses mais ne trompe personne.

    $\arctan(\tan(t))=t$ pour toute valeur de $t$ où on peut calculer $\tan t$.

    (la fonction $\arctan$ est définie sur $\mathbb{R}$ mais pas $\tan$)
  • L'intégrale que j'ai donnée plus haut a pour valeur ,

    $\displaystyle \dfrac{1}{4}\ln^2(1+\sqrt{2})$

    contre toute attente elle ne s'exprime pas en fonction de $\pi$.
  • Bien vu FdP, ça ne marche pas non plus car on retombe sur $\frac{a\arctan(a)}{1+a^4}$.

    Laisse-moi encore un peu chercher avant de donner ta méthode stp, je trouve celle-là assez amusante. :-D
  • Tu as tout le temps que tu veux.
  • @FDP
    bizarrement la méthode de Cyrano ( le début) marche très bien pour $\int_0^{+\infty} \frac{\arctan(x^2)}{1+x^2} dx$ et on trouve une valeur en fonction de $\pi$ ( Wolfram sait le faire aussi)
    Le 😄 Farceur


  • En effet gebrane, l'intégrale jusqu'à l'infini est assez simple en réalité, la borne $1$ de FdP est vicieuse. :-D
  • Si vous savez calculer avec borne infini vous savez calculer avec borne 1 m'enfin ! (désolé pour la mauvaise imitation)

    PS:
    Pas si sûr !!!!
  • On fait un marché , nous on se concentre sur les bornes 0 et 1 et on laisse FDp chercher pourquoi c'est facile pour les bornes 0 et $+\infty$ ;-)
    Le 😄 Farceur


  • Non FDP, on ne sait pas se ramener aux bornes $0,+\infty.$

    Le coup classique c'est d'effectuer le changement de variable $y=1/x$ pour passer de $[0,1]$ à $[1,+\infty]$ puis sommer, sauf qu'il y a un signe "-" qui apparait et qui est embêtant. :-D
  • Le calcul avec la borne infini est simple.
    On se retrouve à additionner deux intégrales dont on ne connait pas la valeur mais on connait la valeur de la somme. :-D

    $\begin{align}J&=\int_0^{\infty} \frac{\arctan\left(x^2\right)}{1+x^2}\,dx\\&=\int_0^{1} \frac{\arctan\left(x^2\right)}{1+x^2}\,dx+\int_1^{\infty} \frac{\arctan\left(x^2\right)}{1+x^2}\,dx\end{align}$

    On fait le changement de variable $\displaystyle y=\frac{1}{x}$ dans la deuxième intégrale.


    $\begin{align}J&=\int_0^{1} \frac{\arctan\left(x^2\right)}{1+x^2}\,dx+\int_0^{1} \frac{\arctan\left(\frac{1}{x^2}\right)}{1+x^2}\,dx\\
    &=\frac{\pi}{2}\int_0^1 \frac{1}{1+x^2}\,dx\\
    &=\frac{\pi^2}{8}
    \end{align}$
  • Cyrano:

    En regardant un peu vite j'ai cru qu'on pouvait y arriver. Mais quand j'ai vu le résultat du calcul sur Wolfram j'ai compris qu'il y avait un problème.
  • Je vais poser ta question sur MSE pour voir
    Le 😄 Farceur


  • Gebrane:

    Ils vont vite trouver ;) (je m'abstiendrai de répondre pour le moment)

    Les deux intégrales sont liées, elles ont un parent commun. La deuxième intégrale je l'avais rencontrée en essayant de calculer une intégrale sans passer par une intégrale à paramètre ou une intégrale double. Ma démarche avait échoué. Je suis retombé dessus par un autre calcul suggéré par quelqu'un dans cette file de messages.
  • Tu as reçu une réponse Gebrane : https://math.stackexchange.com/questions/2556288/how-to-prove-that-int-01-frac-arctanx21x2-dx-dfrac14-ln21

    Au final, la réponse était la même que celle que je proposais. Je n'ai pas eu le courage de tout écrire mais cela fonctionnait bel et bien. :-D
    De manière générale avec les arctangentes, dériver sous le signe s'impose naturellement.
  • Mon calcul utilise le même truc utilisé par Sangchul Lee mais lui complique, à mon humble avis, la fin du calcul avec l'utilisation de la fonction chi.
  • Je trouve très intéressante cette fonction chi, même l'intervenant Sangchul Lee a déclaré sa surprise que cette fonction chi de Legendre s'applique ici.

    @FDP merci pour ta question, ça m'a fait grimper mes points de réputation :-D
    As-tu une autre aussi coriace ;-)
    Le 😄 Farceur


  • Gebrane:

    Je suis étonné qu'il n'ait pas mis ta question off topic.
    Depuis quelques temps sur ME les intégrales proposées avaient peu d'intérêt (pour moi et d'autres j'en suis sûr) cette intégrale a sorti de la léthargie Sangchul Lee.

    PS:
    Il a mauvaise mémoire car il a déjà fait un calcul similaire :

    http://sos440.tistory.com/83

    (c'est son blog ou ancien blog je ne sais)
  • @FDP

    As-tu déjà traité $\int_0^1 \frac{\sin(x-1)}{\ln(x)} dx$?
    Le 😄 Farceur


  • Bonjour,

    L’integrale se calcule bien comme sur MSE. C’est la méthode classique pour se débarrasser d’une fonction arctangente, qui donne une fraction rationnelle après dérivation. Je cherche toujours une méthode plus simple ou plus courte.
    Une intégration par partie pour que l’argument deviennent linéaire et non pas quadratique semble indispensable. Le résultat semble indiquer la piste d’une intégrale double...
  • Gebrane:

    Je n'aime pas beaucoup les intégrales avec un log au dénominateur (il y a de la constante d'Euler dans l'air) Question de goût.

    YvesM:
    Le lien http://sos440.tistory.com/83 donne une méthode qui n'utilise pas la fonction chi mais du dilogarithme.
    Je n'aime pas davantage cette solution et je cherche une solution plus simple.
    C'est toi qui m'a inspiré cette intégrale.
    En faisant le changement de variable $y=\tan x$.
    et on a l'identité,

    $\cos(2x)=\dfrac{1-\tan^2 x}{1+\tan^2 x}$

    Qu'on peut inverser en:

    $\tan^2 (x)=\dfrac{1-\cos(2x)}{1+\cos(2x)}$

    (se rappeler que la fonction $x\rightarrow \frac{1-x}{1+x}$ est involutive)
  • D'une façon ou d'une autre je pense aussi qu'on peut se ramener à une intégrale calculable par résidus. (Selon moi ça reste la méthode la plus jolie pour le calcul d'intégrale) Néanmoins dans le cas, présent ce n'est pas clair du tout.
  • Il reste comment déduire la valeur $\frac{1}{4}\pi^2\sqrt{2}-\sqrt{2}\ln\left(1+\sqrt{2}\right)^2$ de l’intégrale d'avant $\displaystyle \int_0^{\infty} \dfrac{\arctan(x)\sqrt{\sqrt{1+x^2}-1}}{x\sqrt{1+x^2}}\,dx\ $ (avant les autres ;-))
    Le 😄 Farceur


  • Comme indiqué les deux intégrales étaient dérivées de celle-ci:

    $\begin{align} A&=\int_0^1 \frac{\arctan x}{\sqrt{1-x^2}}\,dx\\
    &=\int_0^{\frac{\pi}{2}}\arctan\left(\sin x\right)\,dx\\
    &=\int_0^{\frac{\pi}{2}}\arctan\left(\cos x\right)\,dx\end{align}$

    Si on connait la valeur de cette intégrale alors on sait calculer,

    $\begin{align}J&=\int_0^{\infty} \dfrac{\arctan(x)\sqrt{\sqrt{1+x^2}-1}}{x\sqrt{1+x^2}}\,dx
    \end{align}$

    En effet, si on fait le changement de variable,

    $x=\dfrac{2y}{1-y^2}$,

    $\begin{align}I&=\sqrt{2}\int_0^{1} \dfrac{\arctan\left(\dfrac{2y}{1-y^2}\right)}{\sqrt{1-y^2}}\,dy\\
    &=2\sqrt{2}\int_0^{1} \dfrac{\arctan y}{\sqrt{1-y^2}}\,dy\\
    &=2\sqrt{2}A
    \end{align}$

    PS:
    Cette intégrale a été créée en faisant le chemin à l'envers il n'y a donc rien de miraculeux dans ce changement de variable.
  • $\displaystyle A=\int_0^{1} \dfrac{\arctan x}{\sqrt{1-x^2}}\,dx$

    On définit pour $a\in [0,+\infty[$,

    $\displaystyle F(a)=\int_0^{1} \dfrac{\arctan(ax)}{\sqrt{1-x^2}}\,dx$.

    On vérifie que $F(0)=0$

    et,

    $\begin{align}\lim_{a\rightarrow +\infty} F(a)&=\frac{\pi}{2}\int_0^1 \frac{1}{\sqrt{1-x^2}}\,dx\\
    &=\frac{\pi}{2}\Big[\arcsin x\Big]_0^1\\
    &=\frac{\pi^2}{4}
    \end{align}$

    Par ailleurs,

    $\begin{align} F^\prime(a)&=\int_0^1 \frac{x}{(1+a^2x^2)\sqrt{1-x^2}}\,dx\\
    &=\dfrac{1}{2} \left[\frac{\ln\left(\frac{\sqrt{1+a^2}-a\sqrt{1-x^2}}{\sqrt{1+a^2}+a\sqrt{1-x^2}}\right)}{a\sqrt{1+a^2}}\right]_{x=0}^{x=1}\\
    &=-\dfrac{1}{2}\frac{\ln\left(\frac{\sqrt{1+a^2}-a}{\sqrt{1+a^2}+a}\right)}{a\sqrt{1+a^2}}\\
    &= -\dfrac{1}{2}\frac{\ln\left(\frac{1}{\left(\sqrt{1+a^2}+a\right)^2}\right)}{a\sqrt{1+a^2}}\\
    &=\frac{\ln\left(\sqrt{1+a^2}+a\right)}{a\sqrt{1+a^2}}\\
    &=\frac{\operatorname{arcsinh} a}{a\sqrt{1+a^2}}\\
    \end{align}$

    $\begin{align}A&=F(1)-F(0)\\
    &=\int_0^1 \frac{\operatorname{arcsinh} a}{a\sqrt{1+a^2}}\,da\\
    &=\left[-\operatorname{arcsinh}\left(\frac{1}{a}\right)\operatorname{arcsinh} a\right]_0^1+\int_0^1 \frac{\operatorname{arcsinh}\left(\frac{1}{a}\right)}{\sqrt{1+a^2}}\,da\\
    &=-\ln^2\left(1+\sqrt{2}\right)+\int_0^1 \frac{\operatorname{arcsinh}\left(\frac{1}{a}\right)}{\sqrt{1+a^2}}\,da\\
    \end{align}$

    On fait le changement de variable $x=\dfrac{1}{a}$,

    $\begin{align}A&=-\ln^2\left(1+\sqrt{2}\right)+\int_1^{\infty} \frac{\operatorname{arcsinh}x}{x\sqrt{1+x^2}}\,dx\\
    &=-\ln^2\left(1+\sqrt{2}\right)+\lim_{x\rightarrow +\infty} F(x)-F(1)\\
    &=-\ln^2\left(1+\sqrt{2}\right)+\lim_{x\rightarrow +\infty} \left(F(x)-F(0)\right)-\left(F(1)-F(0)\right)\\
    &=-\ln^2\left(1+\sqrt{2}\right)+\frac{\pi^2}{4}-A\\
    \end{align}$

    Donc,

    $\displaystyle \boxed{A=\dfrac{\pi^2}{8}-\dfrac{1}{2}\ln^2\left(1+\sqrt{2}\right)}$
  • Comment calculeriez-vous cette intégrale ? $$\displaystyle \int_0^1 \frac{1}{5x^4-8x^2+4}\,dx$$
    PS : Le résultat est un nombre réel donc il n'est pas abusif de demander que dans la solution il n'y ait pas de nombre complexe qui y figure.

    Cette intégrale peut intervenir dans le calcul de $\int_{0}^{\pi/2}\arctan\left({2\over \cos^2{x}}\right)\mathrm dx$
    (cf https://math.stackexchange.com/questions/2568911/integral-involving-phi )
  • Bonjour,
    Cette petite factorisation devrait régler le problème sur $\R$ avec les méthodes classiques (écriture pénible...)
    \[
    5 (x^{2} + {\displaystyle \frac {2}{5}} \,\sqrt{10 + 5\,\sqrt{5}}\,
    x + {\displaystyle \frac {2}{5}} \,\sqrt{5})\,(x^{2} -
    {\displaystyle \frac {2}{5}} \,\sqrt{10 + 5\,\sqrt{5}}\,x +
    {\displaystyle \frac {2}{5}} \,\sqrt{5})
    \]

    Cordialement
  • C'est monstrueux, une primitive est \begin{align*}
    ~&\int\dfrac{1}{5\left(x^2-\frac{2\sqrt{\sqrt{5}+2}}{\sqrt{5}}x+\frac{2}{\sqrt{5}}\right)\left(x^2+\frac{2\sqrt{\sqrt{5}+2}}{\sqrt{5}}x+\frac{2}{\sqrt{5}}\right)}\,\mathrm{d}x \\
    &\qquad=\dfrac{\ln\left(\left|x\left(\sqrt{5}x+2\sqrt{\sqrt{5}+2}\right)+2\right|\right)-\ln\left(\left|x\left(\sqrt{5}x-2\sqrt{\sqrt{5}+2}\right)+2\right|\right)}{16\sqrt{\sqrt{5}+2}} \\
    &\qquad\quad+\dfrac{\arctan\left(\frac{\sqrt{5}x+\sqrt{\sqrt{5}+2}}{\sqrt{\sqrt{5}-2}}\right)+\arctan\left(\frac{\sqrt{5}x-\sqrt{\sqrt{5}+2}}{\sqrt{\sqrt{5}-2}}\right)}{8\sqrt{\sqrt{5}-2}}
    \end{align*} J'ai la flemme de faire l’évaluation entre 0 et 1
    Le 😄 Farceur


  • On effectue le changement de variable $x=1/y$ et juste après le changement de variable $u=y-1$. Pour obtenir $$\int_0^1 \frac{1}{5x^4-8x^2+4}\,dx = \int_1^{+\infty} \frac{y^2}{5-8y^2+4y^4} \, dy = \int_0^{+\infty} \frac{(u+1)^2}{4u^4+16u^3+16u^2+1} \, du.$$

    Il suffit après d'appliquer le théorème des résidus à $$\int_C \frac{(z+1)^2 \ln(z)}{4z^4+16z^3+16z^2+1} \, dz,$$ où $C$ est le contour pacman traditionnel. En effet le théorème s'applique aisément car les 4 pôles se trouvent facilement, il s'agit de $-1 \pm \sqrt{1\pm \frac{i}{2}}.$
  • Faire les calculs sans passer par les complexes comme demande FDP est lourd, même si les calculs se simplifient...
    On obtient :

    $I = \displaystyle \int_0^1 \frac{1}{5x^4-8x^2+4}\,dx = \frac {1}{16 \sqrt{a}} \, {\ln\Big( \frac {\sqrt{a} + 2}{\sqrt{a} - 2} \Big)} + \frac {1}{8} \,\sqrt{a}\,\big(\mathrm {Arctan} (a + \sqrt{5a}) - \mathrm {Arctan} (a - \sqrt{5a})\big)$

    Edit : se simplifie encore
    $I = \displaystyle \int_0^1 \frac{1}{5x^4-8x^2+4}\,dx = \frac {1}{16 \sqrt{a}} \, {\ln\Big( \frac {\sqrt{a} + 2}{\sqrt{a} - 2} \Big)} + \frac {1}{8} \,\sqrt{a}\, \Big (\pi -\mathrm {Arctan} ( 2 \sqrt{a}) \Big)$

    avec $a= 2 + \sqrt{5}$


    $I = \displaystyle \int_0^1 \frac{1}{5x^4-8x^2+4}\,dx = \frac {1}{8 \alpha} \, \mathrm{Argth} \Big( \frac{2}{\alpha}\Big) + \frac {\alpha}{8} \Big (\pi - \mathrm {Arctan} ( 2 \alpha) \Big)$

    avec $\alpha = \sqrt{2 + \sqrt{5}}$



    $ I \simeq 0,59434 $


    PS:
    Conformément aux programmes actuels en sup , $\arctan$ redevient $\mathrm {Arctan} $ , notation que l'on n'aurait jamais dû quitter à mon avis...
  • Merci pour ces calculs.

    J'ai commencé par faire le changement de variable fait par Cyrano. Naïvement je croyais qu'il n'y avait pas de terme logarithmique dans le résultat.
  • Je vous propose le calcul (donner une forme close) de l'intégrale suivante:

    $\displaystyle \int_0^1 \frac{\ln(2+x^2)}{1+x^2}\,dx$
  • $\displaystyle \int_0^1 \frac{\ln(2+x^2)}{1+x^2}\,dx=\frac{\pi}{2}\ln(1+\sqrt{2})+\Im\left(\operatorname{Li}_2\Big((3-\sqrt{2})i\right)\Big)-G$

    ($G$ est la constante de Catalan, $\operatorname{Li}_2$ est la fonction dilogarithme)
  • Une intégrale pour le Week-end:

    $\displaystyle \int_0^{\frac{\pi}{2}}\ln\left(1+\sin \theta\right)\ln\left(1+\frac{1}{\sin \theta}\right)\,d\theta$
  • Pendant que je suis là.
    La dernière intégrale vaut $\displaystyle \frac{\pi^3}{16}-2G\ln 2$

    Je n'ai pas trouvé de moyen direct de la calculer. Mon calcul n'est pas difficile conceptuellement mais pénible car (un peu) long.
  • Tu peux nous donner la première étape du calcul ?

    Parce que, juste le résultat, c'est pas hyper évocateur.
  • Tu peux commencer par lire:

    https://math.stackexchange.com/questions/2644467/closed-form-for-the-following-integrals-1-int-01-frac-ln1-x2-ln-1x2/2646505#2646505

    (lis aussi les commentaires)

    Je n'ai pas le temps tout de suite maintenant de donner plus de précision (le lien indiqué donne toutes les réponses si on creuse un tout petit peu)
  • Quelques explications.

    $J=\displaystyle \int_0^{\frac{\pi}{2}}\ln\left(1+\sin \theta\right)\ln\left(1+\frac{1}{\sin \theta}\right)\,d\theta$

    Si on fait le changement de variable $\displaystyle y=\sqrt{\frac{1-\sin \theta}{1+\sin \theta}}$ on obtient,

    $\begin{align}
    J&=2\int_0^1 \ln\left(\frac{2}{1+y^2}\right)\ln\left(\frac{2}{1-y^2}\right)\times \frac{1}{1+y^2}\,dy\\
    &=4G\ln 2-\pi\ln^2 2+2\int_0^1 \frac{\ln(1+y^2)\ln(1-y^2)}{1+y^2}\,dy
    \end{align}$

    Cette dernière intégrale est calculée sur la page mise en lien plus haut.

    Ces intégrales sont souvent combinaisons linéaires à coefficients rationnels des nombres:
    $\displaystyle G\ln 2,\pi^3,\pi\ln^2 2,\zeta(3),\int_0^{\frac{\pi}{4}}\ln^2(\cos x)\,dx$

    L'intégrale est un nombre "moche". Elle ne semble pas pouvoir s'exprimer "simplement". (elle a une expression dans laquelle intervient la partie réelle ou imaginaire d'une fonction polylogarithme avec un argument complexe).

    On peut fabriquer des intégrales comme celle proposée dans laquelle on a neutralisé ce terme par choix de "bons" coefficients (on prend deux telles intégrales pour en fabriquer une troisième. C'est ce qui est fait dans l'intégrale qui est calculée sur la page mise en lien, $\ln(1-x^2)=\ln(1+x)+\ln(1-x)$).

    La difficulté est qu'on ne sait pas toujours exprimer ces intégrales par un calcul direct comme une telle combinaison linéaire.

    Par exemple, si je me souviens bien.

    L'intégrale $\displaystyle \int_0^1 \frac{\ln x\ln(1+x)}{1+x^2}\,dx$ est exprimable comme une telle combinaison linéaire.
    Avec PARI GP et ses fonctions , lindep(),intnum() vous pouvez déterminer empiriquement les coefficients correspondants.

    Mais il n'est pas immédiat de donner une justification à l'égalité pressentie.

    C'est beaucoup plus facile pour l'intégrale,

    $\displaystyle \int_0^1 \frac{\ln^2(1+x^2)}{1+x^2}\,dx$ si je me souviens bien.
  • Une "petite" intégrale pour cette fin de semaine:

    $\displaystyle \int_0^1 \frac{\arctan x}{x}\ln\left(\frac{1+x^2}{(1-x)^2}\right)\,dx$

    C'est le problème 12054 donné dans le dernier American mathematical monthly.
    J'ai déjà une méthode de calcul qui fonctionne bien et je me demande si on peut faire plus simple.

    PS:
    L'énoncé original donne la valeur de l'intégrale. Je vous laisse la trouver.
  • Est-ce que un DSE de arctan(x) est une bonne piste ?

    Autre piste utilisez le DSE de ln(1-t) a ln(1+x^2) - 2ln(1-x) ?

    Une autre changement variable x=exp(-t) ?
  • Pour calculer ce type d'intégrales parfois on essaie par développement en série entière de se ramener au calcul d'une somme d'Euler.
    (cf. http://mathworld.wolfram.com/EulerSum.html pour des exemples de telles sommes)
    Je ne sais pas si l'approche est pertinente dans le cas d'espèce.

    Je pense que j'ai deux méthodes différentes pour calculer cette intégrale. Il me reste à finir un calcul pour être sûr que la deuxième méthode fonctionne. cf. le fil que tu as ouvert pour l'une des méthodes.
Connectez-vous ou Inscrivez-vous pour répondre.
Success message!